1 What do you understand by the term "Environmental determinism"? A. It is a belief that a physical environment affects social and cultural development. B. It is a process which shows how an individual organism adjusts to a gradual change in its environment. C. It is the belief that human behaviour is controlled solely by an individual's physiology. D. It shows that cross-cultural and historical diversity results are not caused due to differences in environment.

2 Which of the following is/are the salient features of Chalukyan administration when compared to the Pallavas and Cholas? 1. The Chalukya administration was highly decentralized unlike that of the Pallavas and the Cholas. 2. The Chalukyas did not maintain maritime power unlike Pallavas and Cholas. Which of the above is/are correct? A. 1 only B. 2 only C. Both 1 and 2 D. None

3 The President has recently assented to the Sugar Cess (Amendment) Bill, 2015 to raise the ceiling of a cess as an excise duty on the production of sugar. The additional cess burden will accrue to the Sugar Development Fund (SDF) and used for 1. Setting up of ethanol plants 2. Setting up of cogeneration power projects 3. Rehabilitation of potentially sick viable sugar undertaking Select the correct answer using the codes below. A. 1 and 2 only B. 2 and 3 only C. 3 only D. 1, 2 and 3

4 The Kepler spacecraft launched by NASA in 2009 was tasked with A. Determining how commonly Earth-like planets occur throughout the Milky Way galaxy B. Probing the surface of exo-planets like Pluto C. Establishing inter-galaxial communication using radio waves D. Understanding the pattern of solar spots from orbiting nearest possible to the Sun

5 Histamine is related to 1. Secretion of acid in stomach 2. Allergy from pollen grains 3. Nasal congestion associated with common cold Select the correct answer using the codes below. A. 1 and 2 only B. 2 and 3 only C. 1 and 3 only D. 1, 2 and 3

6 If the total population of a country is increasing, it necessarily means that 1. Population growth rate is increasing. 2. Total fertility rate is increasing Which of the above is/are correct? A. 1 only B. 2 only C. Both 1 and 2 D. None

7 An age-sex pyramid of a nation must become broader at the base if A. The sex ratio declines. B. There are more females adults than males. C. Men and women have equal population. D. Fertility rate is high.

8 Which of the following argument(s) support biological control of pests and plant diseases? 1. Increased biodiversity furthers plant health. 2. Chemical methods of pest control kill both useful and harmful agents indiscriminately. Which of the above is/are correct? A. 1 only B. 2 only C. Both 1 and 2 D. None

9 Which of the following statements is INCORRECT about fermentation? 1. It can occur only in yeast and bacteria. 2. It must be carried out in an anaerobic environment. Which of the above is/are correct? A. 1 only B. 2 only C. Both 1 and 2 D. None 10 If the proportion of people engaged in primary activities in a nation is high, which of the following conclusions may be appropriately drawn from it? A. The nation is a closed economy. B. Extraction of natural resources is a major activity in the economy C. Levels of urbanizations are very low in the country. D. Large parts of population are illiterate.

11 The "Basic Needs Approach" to human development was initially proposed by the International Labour Organisation (ILO). The approach advocates A. Higher the level of income, the higher is the level of human development B. People should be made participants in development not only its passive recipients. C. Building human capabilities in terms of knowledge and basic capabilities D. Ignoring human choices and fulfilling basic needs of defined sections

12 In anaerobic sludge digesters waste decomposing bacteria produce which of these gases? 1. Methane 2. Hydrogen sulphide 3. Carbon dioxide Select the correct answer using the codes below. A. 1 and 2 only B. 2 and 3 only C. 1 and 3 only D. 1, 2 and 3

13 Which of the following is/are the benefit(s) of "tissue culture"? 1. Possibility of recovery of healthy plants from diseased plants 2. Production of multiples of plants in the absence of seeds Which of the above is/are correct? A. 1 only B. 2 only C. Both 1 and 2 D. None

14 The practices of 'Milpa' in America and 'Ladang' in Indonesia and Malaysia are related to A. Shifting cultivation B. Intensive Mixed farming C. Traditional commercial Livestock Rearing D. Nomadic Herding based on indigenous traditions

15 Zabti or Bandobast system during the reign of Akbar is related to A. Land revenue system B. Annexation of enemy territories C. Mansabdari system D. Welfare of travellers

16 Viticulture or grape cultivation is a speciality of the Mediterranean region. What is in the Mediterranean climate that supports growing grapes? 1. Long growing seasons of moderate to warm temperatures 2. Very low precipitation in grape growing seasons that support grape vegetation Which of the above is correct? A. 1 only B. 2 only C. Both 1 and 2 D. None

17 Which of the following is/are the difference(s) between a free trade area and Customs Union? 1. All restrictions on trade and investment are removed in a free trade area, but not in a customs union. 2. Customs Union has a common external tariff for members, but a free trade area does not. 3. Free trade area does not allow free movement of people, but Customs union does. Select the correct answer using the codes below. A. 2 only B. 2 and 3 only C. 1, 2 and 3 D. None of the above

18 AIDS caused by the Human Immuno deficiency Virus (HIV) can spread by 1. Transfusion of contaminated blood and blood products 2. Infected mother to her child through placenta 3. Touching infected patients Select the correct answer using the codes below. A. 1 and 2 only B. 2 and 3 only C. 1 and 3 only D. 1, 2 and 3 19 In which of the following regions extensive commercial grain cultivation is practised? 1. Interior parts of Southeast Asia 2. Pampas of Argentina 3. European Steppes 4. Amazon Basin Select the correct answer using the codes below. A. 1 and 2 only B. 2, 3 and 4 only C. 1 and 3 only D. 2 and 3 only

20 Which of the following industries is/are footloose industries? 1. Sugar 2. Cotton textiles 3. Vegetable oils Select the correct answer using the codes below. A. 1 and 2 only B. 2 only C. 3 only D. None of the above

21 The culture of wearing masks can be found in 1. Indus Valley civilization 2. Natya Shashtra 3. Pre-historic civilizations Select the correct answer using the codes below. A. 1 and 2 only B. 2 and 3 only C. 3 only D. 1, 2 and 3

22 Highways in Russia are not as important as railways. Why? A. Criss-crossing of rivers across the major transportation zones B. Subsidence zones and weak soil that collapse at high loads C. Magnetic activity is unusual that prevents laying down of metallic roads D. Vast geographical expanse

23 Consider the following statements. 1. The military invasion of Sri Lanka entrusted to his son 2. He was a devout follower of Saivism. 3. He completed the construction of the famous Brihadeeswara temple at Tanjore. 4. His last military achievement was a naval expedition against the Maldive Islands which were conquered. The above refer to? A. I B. Kulottunga I C. Rajaraja I D. Pulakesin II

24 Consider the following comparing Coastal shipping with rail or road transport. 1. Coastal cargo vessels emit more carbon dioxide per tonne of cargo moved. 2. Fuel consumption per tonne of cargo moved is more for coastal shipping. 3. Cost of coast-to-coast transportation is lesser in coastal shipping. Select the correct answer using the codes below. A. 1 and 2 only B. 2 and 3 only C. 3 only D. 1, 2 and 3 only

25 Which of the following is most likely to survive in deep ocean waters? A. Brown Algae B. Red Algae C. Green Algae D. Yellow Algae

26 Consider the following about 'Acquired immunity' in human beings? 1. It is pathogen specific. 2. It is based on memory of disease attack and response. 3. It is present at the time of birth. Select the correct answer using the codes below. A. 1 and 2 only B. 2 and 3 only C. 1 and 3 only D. 1, 2 and 3

27 Use of aspartame is limited to cold foods and soft drinks and not cooked meals because A. It becomes toxic when mixed with greasy substances. B. It is unstable at cooking temperature. C. It is too sweet to be used in normal food items. D. It is a volatile sweetener and needs to be stored at cold temperatures.

28 The Union Government has launched Ganga Gram Yojana. Consider the following about it. 1. Villages situated along the banks of river Ganga will be developed under this scheme. 2. It would be mandatory for every household in the selected villages to have functional toilets. 3. The scheme proposes banning any polluting industry in these villages. Select the correct answer using the codes below. A. 1 and 2 only B. 2 and 3 only C. 1 and 3 only D. 2 only

29 OPEC (Organisation of Petroleum Exporting Countries) consists of 1. Indonesia 2. Venezuela 3. Nigeria 4. Ecuador Select the correct answer using the codes below. A. 1 and 2 only B. 3 and 4 only C. 1, 2, 3 and 4 D. None of the above

30 A nation with negative Balance of payments (BoP) is out of foreign exchange reserves. Which of the following agencies can it approach to avert the BoP crisis? A. United Nations Trusteeship Council (UNTC) B. Alliance for Financial Inclusion (AFI) C. World Trade Organization (WTO) D. None of the above

31 The term "Sub Urbanisation" refers to A. A pattern of urbanization where commercial and residential places are completely separated B. Movement of people from congested urban areas to nearby satellite communities C. Reduction of population in lower tier cities D. Movement of population from urban areas to rural areas

32 Hybrid breed crops in India can be found in 1. Maize 2. Wheat 3. Garden Peas Select the correct answer using the codes below. A. 1 and 2 only B. 2 and 3 only C. 1 and 3 only D. 1, 2 and 3

33 Consider the following about Madhubani paintings and Select the correct answer using the codes below. 1. Traditionally women are not allowed to paint Madhubani paintings. 2. Nature themes are not included. 3. The art has not been officially recognized. A. 1 and 2 only B. 2 and 3 only C. 3 only D. None of the above

34 Single Cell Protein (SCP) can be made available for human consumption by 1. Growing microbes on an industrial scale 2. Genetically modifying plant cells to trigger nutrient production Which of the above is/are correct? A. 1 only B. 2 only C. Both 1 and 2 D. None

35 The Defence Acquisition Council (DAC) has approved major changes in the new Defence Procurement Procedure (DDP) 2016. Some of these changes were based on the recommendations of A. Dhirendra Singh Committee B. Rangarajan Committee C. Kelkar Panel D. Deepak Parekh Committee 36 Consider the following about the tribal community 'Gonds'. 1. They are found only in Southern India. 2. These tribes live only in plains. 3. The Gonds paint their walls with depictions of local flora, fauna and gods. Select the correct answer using the codes below. A. 1 and 2 only B. 2 and 3 only C. 3 only D. None of the above

37 India has decided to become member country of the International Energy Agency - Ocean Energy Systems (IEA-OES). How will it help India? 1. By becoming member of IEA-OES, a power deficit country like India can leverage study of ocean renewable energies. 2. Provide technology to a tropical country like India to harness Ocean Thermal Energy Conversion (OTEC) due to high sea surface temperatures Which of the above is/are correct? A. 1 only B. 2 only C. Both 1 and 2 D. None

38 Consider the following about Chola administration. 1. All lands were carefully surveyed and classified for assessment of revenue. 2. Chola kings undertook royal tours to increase the efficiency of the administration. 3. There were tolls and customs on goods taken from one place to another. 4. There were various kinds of professional taxes. Select the correct answer using the codes below. A. 2 and 3 only B. 1, 2 and 3 only C. 1 and 4 only D. 1, 2, 3 and 4

39 Chhau dance is a genre of A. Indian tribal martial dance B. of C. Devotional puppet dance D. Spiritual invocations to achieve rich harvest

40 Agglomeration Economies between industries are formed due to A. Proximity B. Common raw material C. Common customers D. Common technology

41 "It is made of the leather bag used by villagers to transport water. It is like a basic bagpipe, the national musical instrument of Scotland." The description refers to which musical instrument of India? A. Mashak played by the Dholis of Rajasthan B. Kirla played by the tribes of the North-west C. Khadtaal usually played by the Gurjars D. Manjeeras beaten by the indigenous Santhali communities

42 Organisms of which of the following species may undergo aestivation? 1. Snails 2. Fishes 3. Crocodiles Select the correct answer using the codes below. A. 1 and 2 only B. 2 and 3 only C. 1 and 3 only D. 1, 2 and 3

43 The Battle of Tarain is related to A. The Hindu princes of north India fighting against Muhammad Ghori B. The battle between the Marathas and Rajputs C. Revolt in Gujarati kingdom against Mahmud of Ghazni D. Land battles in Sind between Muslim princes

44 The Union Cabinet has approved the Stand Up India Scheme to A. Promote entrepreneurship among Scheduled Caste/Scheduled Tribes (SC/ST) and Women entrepreneurs B. Provide unemployment allowances to ensure a regular flow of income for the unemployed C. Grant scholarships to women to pursue education of their choice D. Fund the ideas of on-campus university students in India

45 Consider the following about the reign of Firoz Tughlaq. 1. He strictly followed the advice of the ulemas in running the administration. 2. The iqta system was made hereditary. 3. He was the first Sultan to impose irrigation tax. Select the correct answer using the codes below. A. 1 and 2 only B. 2 and 3 only C. 1 and 3 only D. 1, 2 and 3

46 Indiscriminate use of antibiotics causes anti-biotic resistance. This can happen due to 1. Natural selection in bacteria in reaction to anti-biotics 2. Colonization resistance mechanism 3. Attack of clonal bacteria on the host immune system 4. By genetic mutation in the bacteria Select the correct answer using the codes below. A. 1 and 4 only B. 2 and 3 only C. 1 and 3 only D. 1, 2, 3 and 4

47 Same substances can act as an antiseptic as well as disinfectant by 1. Varying its concentration 2. Using it on inanimate surfaces instead of animate biological surfaces Which of the above is/are correct? A. 1 only B. 2 only C. Both 1 and 2 D. None

48 Chemicals are added to food for their preservation, enhancing their appeal, and to add nutritive value in them. Which of the following additives have nutritive value? 1. Fat emulsifiers and stabilising agents 2. Preservatives 3. Food colours Select the correct answer using the codes below. A. 1 and 2 only B. 2 and 3 only C. 1 and 3 only D. None of the above

49 Under the Delhi sultanate muqtis or walis A. Maintained law and order and collected the land revenue B. Managed financial accounts of central treasury C. Lead mercenary military units in special warfares D. Administered village assemblies

50 Which of the following is NOT a valid argument for advocating 'free trade' for a developing economy? A. It results in more efficient allocation of resources internationally. B. It improves competition and reduces domestic monopoly in goods and services. C. It allows exporters to reap economies of scale. D. It creates a level-playing field between developing and developed economies.

51 Desert plants have which of the following adaptations for survival? 1. Stomata arranged in deep pits to minimise water loss through transpiration 2. Special photosynthetic pathway enabling their stomata to remain closed during day time 3. Roots performing photosynthetic functions instead of thorny or no leaves Select the correct answer using the codes below. A. 1 and 2 only B. 2 and 3 only C. 1 only D. 1, 2 and 3

52 Under the WTO agreements, countries cannot normally discriminate between their trading partners– a treatment which is known as “Most favoured Nation (MFN)” status. However some discrimination is allowed when 1. Countries can set up a free trade agreement that applies only to goods traded within the group 2. They can give developing countries special access to their markets 3. A country can raise barriers against products that are considered to be traded unfairly from specific countries Select the correct answer using the codes below. A. 1 and 2 only B. 2 and 3 only C. 1 and 3 only D. 1, 2 and 3

53 Which one of the following types of cultivation was developed by European colonists in India? 1. Kolkoz 2. Mixed farming 3. Plantation Select the correct answer using the codes below. A. 1 and 2 only B. 2 and 3 only C. 3 only D. 1, 2 and 3

54 Consider the following statements. Assertion (A): Consuming carcinogenic substances consistently for long durations may result in cell tumours. Reason (R): In cancer, cell growth and differentiation becomes uncontrolled and difficult to regulate by the body. In the context of the above, which of these is correct? A. A is correct, and R can be an appropriate explanation of A. B. A is correct, but R cannot be an appropriate explanation of A. C. A is correct, but R is incorrect. D. Both A and R are incorrect.

55 Which of following criteria is most useful to differentiate an Indian urban area from a rural area? A. Literacy rates B. Occupational structure C. Sex ratio D. Growth rate of population

56 Biofortification refers to A. Breeding crops with higher levels of nutrition B. Enabling the plants to bear higher abiotic stresses C. Reducing the water requirement for the crops D. Increasing the physical strength of the plant

57 Which of the following may be a valid reason for artistic and respectful representations of many snakes in traditional paintings like Gondi? A. Snakes were considered a spiritual symbol and showed man in harmony with nature. B. The paintings were used to create awareness about vituperous snakes. C. Snakes were killed on a large scale in several rituals and festivals which were depicted in the traditional paintings. D. Snakes were painted to get divine blessings before major warfare.

58 The practice of 'transhumance' refers to A. Seasonal migration in between plains and mountains in search for pasture B. Adjusting livestock in the field according to the carrying capacity of the pastureland C. Rearing of animals on mountain ranges based on scientific practices D. Import and export of cattle by local community for improving pastureland

59 Proportion of literate population of a country known as literacy in an indicator of its socio-economic development. Literacy in India will necessarily increase if A. Total years of schooling for the entire population increase B. Sanitation and health facilities are improved C. Economic growth rate is high D. More children pass the primary school in the country

60 The sewage water is treated untill the A. Biochemical oxygen demand (BOD) of the sewage water increases significantly B. Chemical oxygen demand (COD) of the sewage water increases significantly C. Chemical oxygen demand (COD) of the sewage water is reduced significantly D. Biochemical oxygen demand (BOD) of the sewage water is reduced significantly

61 Gross National Happiness (GNH) is Bhutan's measure of national progress. Which of the following indicators is/are common to both GNH and Human Development Index (HDI)? 1. Sustainable development 2. Establishment of good governance 3. Preservation and promotion of cultural values Select the correct answer using the codes below. A. 1 and 2 only B. 2 and 3 only C. 1 only D. None of the above

62 Ganga Action Plan (GAP) and Yamuna Action Plan (YAP) were initiated before the Namami Ganga programme. GAP and YAP were initiated by the A. Ministry of Environment and Forests B. Ministry of Drinking Water and Sanitation C. Ministry of Planning D. Ministry of Culture

63 Which of the following terms connote the opposite meaning of the term "Population aging"? A. Sub-replacement fertility B. Pensions crisis C. Demographic trap D. Demographic Dividend

64 Which of these can be used as a bio-pesticide? 1. Bacteria 2. Viruses 3. Fungi 4. Insects Select the correct answer using the codes below. A. 1, 2 and 4 only B. 2 and 3 only C. 1 and 4 only D. 1, 2, 3 and 4

65 China started being influenced by Indian culture from 1st A.D due to which of the following? 1. Land route passing through Central Asia 2. Sea route through Burma (Myanmar) Which of the above is/are correct? A. 1 only B. 2 only C. Both 1 and 2 D. None

66 Genetic modification techniques offer which of the following advantage(s) as compared to the traditional hybridisation technique? 1. Genetic modification technique may not lead to inclusion and multiplication of undesirable genes along with the desired genes. 2. Genetic modification technique gives precise and quicker results unlike the traditional technique. Which of the above is/are correct? A. 1 only B. 2 only C. Both 1 and 2 D. None

67 The jharnapatachitra of West Bengal is a(an) A. Long vertical paper scroll used to tell stories from religious epics B. Devotional stanzas by ancient poets inscribed on rocks C. Inner wall decoration of elite tribal homes D. Expression of prevention of child mortality by showing how elephants and other animals protect their young ones

68 Which of the following regions is likely to have the highest mean annual temperature as well as the highest mean annual precipitation? A. Coniferous forest B. Grassland C. Tropical forest D. Temperate forest

69 X is one of the neurotransmitters that play a role in mood changes. If the level of X is low for some reason, then the signal- sending activity becomes low, and the person suffers from depression. 'X' here may refer to A. Seldane B. Rantidine C. Noradrenaline D. Acetylcholine

70 Many freshwater animals cannot live for long in sea water and many sea water animals also cannot live for long in freshwater. Which of these can be a possible factor? A. Osmosis across body membrane of the fish may kill it. B. Sea water is heavier and crushes the skin of the freshwater fish, whereas freshwater makes sea water fish skin succulent C. Level of dissolved oxygen varies causing respiration problems. D. The dissolved nutrients in water are a major factor in determining the longevity of the fish.

71 Which of the following canal connects the Atlantic Ocean in the east to the Pacific Ocean in the west? A. Suez Canal B. Panama Canal C. Hormuz Canal D. Vistok Canal

72 Consider these matches of popular musical instruments with the region they belong to. 1. Pakhawaj : North-Eastern India 2. Maddalam : Southern India 3. Thumbak nari : Northern India Which of the above is/are correct? A. 1 and 2 only B. 2 and 3 only C. 1 and 3 only D. 1, 2 and 3

73 Which of the following problems may be caused due to use of detergents? 1. Bacteria may not be able to degrade them. 2. These persist in water even after sewage treatment and cause foaming in rivers. Which of the above is/are correct? A. 1 only B. 2 only C. Both 1 and 2 D. None

74 Interspecific interactions can have many outcomes. Which of the following is an example of 'mutualism' interaction? A. Bees sucking nectar from flowers and helping them in pollination B. An orchid growing on top of a mango branch C. Mosquitoes sucking blood from its prey and infecting it with vector-borne diseases D. Competition between tigers and lions for prey in African forests

75 is known as the Cradle of Indian architecture. The Aihole inscription was issued under the reign of A. Chalukyas B. Pallavas C. Rashtrakutas D. Marathas

76 A small increase in population is desirable in a growing economy. This is because A. Lower population growth shows higher levels of economic development. B. A growing economy requires growing labour force. C. Human capital builds natural capital. D. Higher increase in population leads to loss of human capital.

77 Under the reign of Rashtrakutas 1. The village administration was carried on by the village headmen. 2. Officers administering districts or subdivisions were directly appointed by the central government. Which of the above is/are correct? A. 1 only B. 2 only C. Both 1 and 2 D. None

78 Consider the following statements. Assertion (A): The Northern Atlantic Sea Route is one of the busiest in the world. Reason (R): It is only route with natural harbours amongst all trade routes. In the context of the above, which of these is correct? A. A is correct, and R can be an appropriate explanation of A. B. A is correct, but R cannot be an appropriate explanation of A. C. A is correct, but R is incorrect. D. Both A and R are incorrect.

79 Consider the following about bioreactors. 1. It can support both aerobic and anaerobic processes. 2. It can be used to grow cells or tissues. 3. Bioreactors must be left open to allow biological reactions to take place. Select the correct answer using the codes below. A. 1 and 2 only B. 2 and 3 only C. 1 only D. 1, 2 and 3

80 Consider the following statements about Cyanobacteria. 1. They are found only in aquatic environments. 2. They can serve as a biofertiliser in paddy fields. 3. They add organic matter to the soil and increase its fertility. Select the correct answer using the codes below. A. 1 and 2 only B. 2 and 3 only C. 1 and 3 only D. 1, 2 and 3

81 One of your office colleagues is suffering from severe irritation and pain due to acidity in the stomach. You would recommend Antacids to her because A. It cures Ulcer which may be the cause of irritation and pain B. It reduces formation of peptacides and bile juice in the stomach C. It reduces the pH of the stomach which relieves the pain D. None of the above

82 In his reign, the introduction of paying salaries in cash to the soldiers led to price regulations popularly called as Market Reforms. He was the first Sultan of Delhi who ordered for the measurement of land. His land revenue reforms provided a basis for the future reforms of Sher Shah and Akbar. He is? A. Alauddin Khalji B. Ghazi Malik C. Balban D. Iltutmish

83 Consider the following statements about certain species and their biome habitats. 1. Mango trees are grown in temperate countries also alongwith tropical countries. 2. Tune fish cannot be found in tropical latitudes in the ocean. 3. Snow leopards can be found in Kerala . Select the correct answer using the codes below. A. 1 and 2 only B. 2 and 3 only C. 1 and 3 only D. None of the above

84 A department called Diwan-i-Khairat was created under the reign of Firoz Tughlaq to A. Consolidate decentralized treasuries across the empire B. Take care of orphans and widows C. Address public grievances and complaints against officials D. Take care of religious tourists and travellers

85 Which of the following is/are artificial sweeteners? 1. Alitame 2. Sucrolose 3. Saccharin 4. Lactamase Select the correct answer using the codes below. A. 2 and 4 only B. 1, 2 and 3 only C. 3 and 4 only D. 1, 2, 3 and 4

86 Consider the following about beliefs of Sufis. 1. Love and devotion is a means of attaining salvation. 2. One must have the guidance of a master without which spiritual development is impossible. Which of the above is/are correct? A. 1 only B. 2 only C. Both 1 and 2 D. None

87 The Union Cabinet has recently approved Pradhan Mantri Fasal Bima Yojana. How will it help the farmers? 1. Reduce the premium burden on farmers 2. Ensure early settlement of crop assurance claim Which of the above is/are correct? A. 1 only B. 2 only C. Both 1 and 2 D. None

88 Wall paintings can be found in 1. Buddhist viharas in Ajanta 2. Prehistoric paintings in rock shelters 3. Virubhadra Temple in Lepakshi Select the correct answer using the codes below. A. 1 and 2 only B. 2 and 3 only C. 3 only D. 1, 2 and 3

89 The first Indian state to achieve 100 per cent primary education is A. Mizoram B. Himachal Pradesh C. Kerala D. Punjab

90 As per the Democratic transition theory 1. High levels of birth rates in a society are achieved at the last stage of the transition. 2. Low levels of death rates in a society are achieved at the earliest stage of the transition. Which of the above is/are correct? A. 1 only B. 2 only C. Both 1 and 2 D. None

91 Consider the following about the Employee Provident Fund Organisation (EPFO). 1. EPFO is a statutory body coming under the aegis of Ministry of Labour and Employment. 2. It administers the Pension Scheme and Insurance Scheme. Which of the above is/are correct? A. 1 only B. 2 only C. Both 1 and 2 D. None

92 The India Meteorological Department (IMD) has recently re-defined terms that are not scientifically precise in weather forecasting. As per the new definitions, a "Large Deficient year" for rainfall refers to A. Metrological drought in at least a majority of states of India B. Rainfall deficit of over 10 per cent across more than 40 per cent of India's area C. Rainfall less than half of the average annual precipitation over India as a whole D. Hydrological drought in major basins of India where rainfall is less than the average annual precipitation

93 The "Gajendra Moksham" painting in the Krishnapuram Palace near Kayamkulam is an example of A. Mural painting B. Rock cut painting C. Fusion of Persian Miniature and Western painting D. Contextual Modernist painting

94 Consider the following statements about India's demographics. 1. Majority of India's population is below the age of 25. 2. The national Census of India does not recognise racial groups within India. 3. Population density is highest in the state of Madhya Pradesh. Select the correct answer using the codes below. A. 1 and 2 only B. 3 only C. 2 only D. 1 and 3 only

95 The President has promulgated the Enemy Property (Amendment and Validation) Ordinance, 2016 to make amendments to the Enemy Property Act, 1968. Which of the following agreements is relevant to these existing laws? A. Tashkent Declaration (1966) B. Panchsheel Declaration– India and China C. Simla Agreement (1971) D. Simla Accord (1914)

96 The Rate of Natural Increase (RNI) of population includes 1. Immigration 2. Death rates 3. Emigration Select the correct answer using the codes below. A. 1 and 2 only B. 3 only C. 2 only D. 1 and 3 only

97 Consider the following about regional trading blocs. 1. A trading bloc can only be formed when all tariff and non-tariff restrictions are removed between members. 2. A trading bloc adopts a common economic policy. Which of the above is/are correct? A. 1 only B. 2 only C. Both 1 and 2 D. None

98 The Union Government has abolished export duty on iron ore pellets in order to make the commodity more competitive amid weakening prices.What factor(s) can be said responsible behind the weakening of prices? 1. Global economic slowdown has caused slackening of demand and thus prices 2. Subdued demand for steel and related raw materials in USA which is world's largest metal consumer Which of the above is/are correct? A. 1 only B. 2 only C. Both 1 and 2 D. None

99 Which of the following is/are the major recommendations of the Lodha Committee? 1. Not Ministers but government servants should be allowed to becomeBCCI officebearers. 2. Betting should be legalised and match fixing should be made criminal offence. 3. BCCI should come under the purview of the Right to Information Act, 2005. Select the correct answer using the codes below. A. 1 and 2 only B. 2 and 3 only C. 1 and 3 only D. 1, 2 and 3

100 Which of the following statements about the administration under the Vijayanagar Empire is INCORRECT? A. The king enjoyed absolute authority in executive, judicial and legislative matters. B. The King was the highest court of appeal. C. The succession to the throne was on the principle of merit. D. The king was assisted by a council of ministers in his day to day administration.

1 Correct Answer : A Justification: Option (b) is similar to "Acclimatization". Option (c) is Biological determinism. The most appropriate option is (a), as it is the belief that the physical environment predisposes human social development towards particular trajectories. Learning: The fundamental argument of the environmental determinists was that aspects of physical geography, particularly climate, influenced the psychological mind-set of individuals, which in turn defined the behaviour and culture of the society that those individuals formed. For e.g., tropical climates were said to cause laziness, relaxed attitudes, promiscuity and generally degenerative societies, while the frequent variability in the weather of the middle latitudes led to more determined and driven work ethics and thus more civilized and 'stronger' societies. The theory that opposes it is Possibilism. It argues that the environment only sets certain constraints or limitations, but culture is otherwise determined by social conditions. Neodeterminism or stop and go determinism is the middle way between both concepts. Q Source: Page 3: Chapter 1: NCERT XIIth: Fundamentals of Human Geography

2 Correct Answer : D Justification: The Chalukya administration was highly centralized unlike that of the Pallavas and the Cholas. Village autonomy was absent under the Chalukyas. So, 1 is incorrect. The system of village autonomy with sabhas and their committees developed through the ages and reached its culmination during the Chola rule. The Chalukyas had a great maritime power. Pulakesin II had 100 ships in his navy. They also had a small standing army. So, 2 is incorrect. The Cholas paid special attention to their navy. The naval achievements of the reached its climax under the Cholas. Q Source: Page 132: Chapter 12: Class XIth: TamilNadu History Textbook

3 Correct Answer : D Justification: Sugar Development Fund (SDF) can be used for: Development of cane in their areas; Rehabilitation/Modernisation projects; Setting up of ethanol plants; Setting up of cogeneration power projects; Rehabilitation of potentially sick viable sugar undertaking etc. Cane growers can apply for a loan from the SDF for these projects. The Bill proposes to increase this ceiling of sugar to Rs 200 per quintal from existing Rs 25 per quintal specified in the principal Act. The principal Act empowers central government to fix the rate of sugar cess from time to time. Implications of hike in sugar cess: It would lead to increase in the accruals to the Sugar Development Fund (SDF) which in turn would facilitate rehabilitation and modernisation of sugar mills and thereby helping cane growers. In present scenario, it will help in revival of stagnant sugar economy by helping cash-starved mills and sugarcane growers. Q Source: http://pib.nic.in/newsite/PrintRelease.aspx?relid=133115 http://dfpd.nic.in/allocation-of-work.htm

4 Correct Answer : A Learning: The National Aeronautics and Space Administration (NASA's) revamped planet-hunting Kepler spacecraft has discovered more than 100 planets orbiting other stars. The Kepler spacecraft finds the planets by using the transit method. It is detection of the tiny brightness dips caused by the planet after it crosses its host star's face from spacecraft's perspective. Transit method technique requires extremely precise pointing of spacecraft. Q Source: http://indianexpress.com/article/technology/science/the-search-for-another-earth/ 5 Correct Answer : D Justification: Statement 1: Histamine stimulates the secretion of pepsin and hydrochloric acid in the stomach. A drug cimetidine (Tegamet), was designed to prevent the interaction of histamine with the receptors present in the stomach wall. This resulted in release of lesser amount of acid. The importance of the drug was so much that it remained the largest selling drug in the world until another drug, ranitidine (Zantac), was discovered. Statement 2 & 3 and additional info: Histamine is a potent vasodilator. It has various functions. It contracts the smooth muscles in the bronchi and gut and relaxes other muscles, such as those in the walls of fine blood vessels. Histamine is also responsible for the nasal congestion associated with common cold and allergic response to pollen. Q Source: Page 443: Unit 16: NCERT XIIth: Chemistry

6 Correct Answer : D Justification: Growth rate need not increase. Even if it is constant or negative, total population can grow. For e.g. if 2013 population grew at 5%, in 2014 at 4%, even then total population increases. So, 1 is wrong. Population may increase due to immigration and low death rates too. So, 2 is not necessarily correct. Q Source: Page 15: Chapter 2: NCERT XIIth: Fundamentals of Human Geography

7 Correct Answer : D Justification: Even if sex-ratio declines, if total child population is constant, pyramid will not flatten at the base. So, (a) is incorrect. Base shows child population, not adult population. So, (b) is wrong. If men and women have equal population for all ages, the pyramid may shows vertically symmetry. It will not flatten the base. So, (c) is wrong. Q Source: Page 18: Chapter 3: NCERT XIIth: Fundamentals of Human Geography

8 Correct Answer : C Justification & Learning: In agriculture, there is a method of controlling pests that relies on natural predation rather than introduced chemicals. A key belief of the organic farmer is that biodiversity furthers health. The more variety a landscape has, the more sustainable it is. So, 1 is correct. The organic farmer, therefore, works to create a system where the insects that are sometimes called pests are not eradicated, but instead are kept at manageable levels by a complex system of checks and balances within a living and vibrant ecosystem. The organic farmer holds the view that the eradication of the creatures that are often described as pests is not only possible, but also undesirable, for without them the beneficial predatory and parasitic insects which depend upon them as food or hosts would not be able to survive. So, 2 is correct. Q Source: Page 187: Chapter 10: NCERT XIIth: Biology

9 Correct Answer : C Justification: Statement 1: Fermentation is a metabolic process that converts sugar to acids, gases or alcohol. It occurs in yeast and bacteria, and also in oxygen-starved muscle cells, as in the case of lactic acid fermentation. So, 1 is wrong. Statement 2: Fermentation does not necessarily have to be carried out in an anaerobic environment. For example, even in the presence of abundant oxygen, yeast cells greatly prefer fermentation to aerobic respiration, as long as sugars are readily available for consumption. So, 2 is also wrong. Q Source: Improvisation: Page 182: Chapter 10: NCERT XIIth: Biology

10 Correct Answer : B Justification: A nation can be a closed or an open economy. For e.g. in India the number is high at around 50%, but it is an open economy. (a) is wrong. Primary activities involve Extraction of natural resources like mining, agriculture etc. So, (b) is correct. In India urbanization stands at 32%, yet 50% or more work in primary sector. (c) is not correct. Option (d) need not be true. India is a classic example. Q Source: Page 20: Chapter 3: NCERT XIIth: Fundamentals of Human Geography

11 Correct Answer : D Learning: Basic needs approaches to well-being are concerned with outcomes. They arose out of recognition that economic approaches to development that focused on incomes and growth were problematic. A narrow economic understanding sees increasing incomes and faster growth as proxies for improved well-being. However, basic needs proponents argue that this depends, firstly, on whether income is spent on satisfying basic needs where there are shortfalls or on luxuries that may ultimately have a detrimental impact on well-being and, secondly, on how income and the positive welfare benefits of increased income are distributed between and within different social groups. Basic needs approaches focus on key indicators that are seen as objective assessments of well-being such as longevity, infant survival, body mass index, educational attainment, and so on. Q Source: Page 26: Chapter 4: NCERT XIIth: Fundamentals of Human Geography

12 Correct Answer : D Learning: Anaerobic digestion is particularly suited to organic material, and is commonly used for industrial effluent, wastewater and sewage sludge treatment. Anaerobic digestion, a simple process, can greatly reduce the amount of organic matter which might otherwise be destined to be dumped at sea, dumped in landfills, or burnt in incinerators. Using anaerobic digestion technologies can help to reduce the emission of greenhouse gases in a number of key ways: Replacement of fossil fuels Reducing or eliminating the energy footprint of waste treatment plants Reducing methane emission from landfills Displacing industrially produced chemical fertilizers Reducing vehicle movements Reducing electrical grid transportation losses Reducing usage of LP Gas for cooking Q Source: Page 184: Chapter 10: NCERT XIIth: Biology

13 Correct Answer : C Learning: It is a collection of techniques used to maintain or grow plant cells, tissues or organs under sterile conditions on a nutrient culture medium of known composition. It is widely used to produce clones of a plant in a method known as micropropagation. Advantages over traditional methods of propagation: The production of exact copies of plants that produce particularly good flowers, fruits, or have other desirable traits. To quickly produce mature plants. The production of multiples of plants in the absence of seeds or necessary pollinators to produce seeds. The regeneration of whole plants from plant cells that have been genetically modified. The production of plants in sterile containers that allows them to be moved with greatly reduced chances of transmitting diseases, pests, and pathogens. The production of plants from seeds that otherwise have very low chances of germinating and growing, i.e.: orchids and Nepenthes. To clean particular plants of viral and other infections and to quickly multiply these plants as 'cleaned stock' for horticulture and agriculture. Plant tissue culture relies on the fact that many plant cells have the ability to regenerate a whole plant (totipotency). Q Source: Page 177: Chapter 10: NCERT XIIth: Biology 14 Correct Answer : A Learning: The vegetation is usually cleared by fire, and the ashes add to the fertility of the soil. Shifting cultivation is thus, also called slash and burn agriculture. The cultivated patches are very small and cultivation is done with very primitive tools such as sticks and hoes. After sometime (3 to 5 years) the soil looses its fertility and the farmer shifts to another parts and clears other patch of the forest for cultivation. The farmer may return to the earlier patch after sometime. One of the major problems of shifting cultivation is that the cycle of jhum becomes less and less due to loss of fertility in different parcels. It is prevalent in tropical region in different names, e.g. Jhuming in North eastern states of India, Milpa in central America and Mexico and Ladang in Indonesia and Malaysia. Q Source: Page 36: Chapter 5: NCERT XIIth: Fundamentals of Human Geography

15 Correct Answer : A Learning: The land revenue system of Akbar was called Zabti or Bandobast system. It was further improved by Raja Todar Mal. It was known as Dahsala System which was completed in 1580. By this system, Todar Mal introduced a uniform system of land measurement. The revenue was fixed on the average yield of land assessed on the basis of past ten years. The land was also divided into four categories – Polaj (cultivated every year), Parauti (once in two years), Chachar (once in three or four years) and Banjar (once in five or more years). Payment of revenue was made generally in cash. Q Source: Page 229: Chapter 20: NCERT XIth: TamilNadu History Textbook

16 Correct Answer : A Justification: The presence of adequate sun, heat and water are all vital to the healthy growth and development of grapevines during the growing season. In general, grapevines thrive in temperate climates which grant the vines long, warm periods during the crucial flowering, fruit set and ripening periods. So, 1 is correct. On average, a grapevine needs around 710 mm (28 in) of water for sustenance during the growing season, not all of which may be provided by natural rain fall. In Mediterranean and many continental climates, the climate during the growing season may be quite dry and require additional irrigation. So, 2 is incorrect. However, the advantage of Mediterranean agriculture is that more valuable crops such as fruits and vegetables are grown in winters when there is great demand in European and North American markets Q Source: Page 40: Chapter 5: NCERT XIIth: Fundamentals of Human Geography

17 Correct Answer : A Justification & Learning: The degree of economic integration can be categorized into seven stages: Preferential trading area Free trade area Customs union Common market Economic union Economic and monetary union Complete economic integration These differ in the degree of unification of economic policies, with the highest one being the completed economic integration of the states, which would most likely involve political integration as well. A "free trade area" (FTA) is formed when at least two states partially or fully abolish custom tariffs on their inner border. However, this may not include complete abolition on investment flow. So, 1 is incorrect. A "customs union" introduces unified tariffs on the exterior borders of the union (CET, common external tariffs). A "monetary union" introduces a shared currency. A "common market" add to a FTA the free movement of services, capital and labor (not a Customs Union). So, 3 is incorrect. An "economic union" combines customs union with a common market. A "fiscal union" introduces a shared fiscal and budgetary policy. Q Source: Improvisation: Page 85: Chapter 9: NCERT XIIth: Fundamentals of Human Geography

18 Correct Answer : A Justification & Learning: HIV/AIDS is not spread by mere touch or physical contact; it spreads only through body fluids. So, 3 is incorrect. Transmission of HIV-infection generally occurs by (a) sexual contact with infected person, (b) by transfusion of contaminated blood and blood products, (c) by sharing infected needles as in the case of intravenous drug abusers and (d) from infected mother to her child through placenta. So, 1 and 2 are correct. So, people who are at high risk of getting this infection includes - individuals who have multiple sexual partners, drug addicts who take drugs intravenously, individuals who require repeated blood transfusions and children born to an HIV infected mother. Q Source: Page 155: Chapter 8: NCERT XIIth: Biology

19 Correct Answer : D Justification: Statement 1 and 4: Gathering is practised in: (i) high latitude zones which include northern Canada, northern Eurasia and southern Chile; (ii) Low latitude zones such as the Amazon Basin, tropical Africa, Northern fringe of Australia and the interior parts of Southeast Asia. Statement 2 and 3: Extensive commercial grain cultivation is best developed in Eurasian steppes, the Canadian and American Prairies, the Pampas of Argentina, the Velds of South Africa, the Australian Downs and the Canterbury Plains of New Zealand. Q Source: Page 38: Chapter 5: NCERT XIIth: Fundamentals of Human Geography

20 Correct Answer : D Justification & Learning: Foot loose industries can be located in a wide variety of places. They are not dependent on any specific raw material, weight losing or otherwise. Since cotton, oil and sugar industries require raw materials and cannot be established anywhere, 1, 2 and 3 all are wrong. They largely depend on component parts which can be obtained anywhere. They produce in small quantity and also employ a small labour force. These are generally not polluting industries. The important factor in their location is accessibility by road network. Q Source: Page 47: Chapter 6: NCERT XIIth: Fundamentals of Human Geography

21 Correct Answer : D JustificationStatement 1: Excavations have revealed small hollow masks dating back to the Indus Valley Civilisation. In fact in Bihar a terracotta mask of the fourth century has also been excavated. Statement 2: The Natya Shastra speaks of masks and their use in theatre. Here it is mentioned that masks can be made of ground paddy husks applied to cloth. Statement 3: The earliest traces of human existence go back to the period between 3,00,000 and 2,00,000 BC. The modern human being first appeared around 36000 BC. The origin of the mask has been traced to pre-historic man. Depictions of masks have been found in various rock paintings and cave paintings. Masks were probably used for hunting and taming animals and for their primeval dances. Q Source:Page 115: Chapter 10: NCERT XIth: Living Craft Traditions of India

22 Correct Answer : D Justification:If option (a) was correct, it should have been logically correct for railways too. So, it can't be the answer. Same is true for option (b). Option (c) is absurd. In Russia, a dense highway network is developed in the industrialised region west of the Urals with Moscow as the hub. The important Moscow-Vladivostok Highway serves the region to the east. Due to the vast geographical area, highways in Russia are not as important as railways. Q Source: Page 68: Chapter 8: NCERT XIIth: Fundamentals of Human Geography

23 Correct Answer : C Learning: It was under Rajaraja I and his son Rajendra I that the Chola power reached its highest point of glory. By his military conquests, the extent of the Chola empire under Rajaraja I included the Pandya, Chera and the Tondaimandalam regions of Tamil Nadu and the Gangavadi, Nolambapadi and the Telugu Choda territories in the Deccan and the northern part of Ceylon and the Maldive Islands beyond India. He completed the construction of the famous Rajarajeswara temple or Brihadeeswara temple at Tanjore in 1010 A.D. He also helped in the construction of a Buddhist monastery at Nagapattinam. Q Source: Page 142: Chapter 13: Class XIth: TamilNadu History Textbook

24 Correct Answer : C Justification:Refer to the image below. In India, coastal shipping moves just 7% of the local freight despite a coastline of 7,517km dotted with 12 state-owned ports and around 200 smaller harbours. The share is low compared with the European Union, where coastal shipping has a 43% share of the cargo traffic. In the US, it has a 15% share. If you can't access the image, refer to page 32 of http://shipmin.nic.in/showfile.php?lid=1959 Q Source: Improvisation: Page 74: Chapter 8: NCERT XIIth: Fundamentals of Human Geography

25 Correct Answer : B Concept: Light is a limiting factor for life in deep oceans. Not all the colour components of the visible spectrum are available for marine plants living at different depths of the ocean. Sunlight is broken into its seven component colours as it penetrates through the ocean water by dispersion. Longer wavelengths cannot penetrate deeper whereas shorter wavelengths do. Justification & Learning:Algae use photosynthesis to manufacture food. So, the ones utilizing the short wavelength for photosynthesis survive. Red algae have red pigment (phycoerythrin) which carries out photosynthesis using only short wavelength light found in greater depths. They can be found as deep as 200 m in ocean water. Q Source: Page 223: Chapter 13: NCERT XIIth: Biology

26 Correct Answer : A Justification: Statement 1 and 2: This means that our body when it encounters a pathogen for the first time produces a response called primary response which is of low intensity. Subsequent encounter with the same pathogen elicits a highly intensified secondary or anamnestic response. This is ascribed to the fact that our body appears to have memory of the first encounter. So, 1 and 2 are correct. Statement 3: It is "innate immunity" which is present at time of birth. It prevents the entry of the foreign agents into our body when our bodies are weak. Q Source:Page 150-151: Chapter 8: NCERT XIIth: Biology

27 Correct Answer : B Learning: Aspartame is one of the most common artificial sweeteners in use today Aspartame is used in many foods and beverages because it is about 100-200 times sweeter than sugar, so much less of it can be used to give the same level of sweetness. This, in turn, lowers the calories in the food or beverage. In the body, aspartame is broken down into phenylalanine, aspartic acid, and methanol. Methanol can be toxic in high amounts, but the amounts that result from the breakdown of aspartame is lower than with many "natural" foods. Q Source: Page 450: Unit 16: NCERT XIIth: Chemistry

28 Correct Answer : A Learning: The Union Government has launched Ganga Gram Yojana to develop 1600 villages in Uttar Pradesh (UP) situated along the banks of Ganga River. In these selected villages open drains discharging waste directly into river Ganga will be diverted to sewage treatment facility. Government will spend one crore rupees on every village under this programme. It would be mandatory for every household in selected villages to have functional toilets. Under this programme, these selected villages will be developed under the Sichewal model (Village in Punjab) which is based on cooperation of the villagers for water management and waste disposal. Q Source: http://pib.nic.in/newsite/PrintRelease.aspx?relid=134183

29 Correct Answer : C Learning:The OPEC Statute distinguishes between the Founder Members and Full Members - those countries whose applications for membership have been accepted by the Conference. The Statute stipulates that "any country with a substantial net export of crude petroleum, which has fundamentally similar interests to those of Member Countries, may become a Full Member of the Organization, if accepted by a majority of threefourths of Full Members, including the concurring votes of all Founder Members." So, even countries outside the middle-east like Indonesia can become members. Q Source: Page 87: Chapter 9: NCERT XIIth: Fundamentals of Human Geography

30 Correct Answer : D Learning: The answer will be International Monetary Fund (IMF). The organization's objectives stated in the Articles of Agreement are:[6] to promote international monetary cooperation, international trade, high employment, exchange-rate stability, sustainable economic growth, and making resources available to member countries in financial difficulty. Member countries of the IMF have access to information on the economic policies of all member countries, the opportunity to influence other members' economic policies, technical assistance in banking, fiscal affairs, and exchange matters, financial support in times of payment difficulties, and increased opportunities for trade and investment Q Source: Improvisation: Chapter 9: NCERT XIIth: Fundamentals of Human Geography

31 Correct Answer : B Concept: It is a new trend of people moving away from congested urban areas to cleaner areas outside the city in search of a better quality of living. Important suburbs develop around major cities and everyday thousands of people commute from their homes in the sub urbs to their work places in the city Justification:Option (a) is somewhat similar (but not same) to commuter towns. Option (c) is a trend of concentration in metropolitan cities. Option (d) is called counter-urbanization. Q Source: Page 91: Chapter 10: NCERT XIIth: Fundamentals of Human Geography

32 Correct Answer : D Learning: Wheat and Rice: During the period 1960 to 2000, wheat production increased from 11 million tones to 75 million tonnes while rice production went up from 35 million tonnes to 89.5 million tonnes. This was due to the development of semi-dwarf varieties of wheat and rice. Sugar cane: Saccharum barberi was originally grown in north India, but had poor sugar content and yield. Tropical canes grown in south India Saccharum officinarum had thicker stems and higher sugar content but did not grow well in north India. These two species were successfully crossed to get sugar cane varieties combining the desirable qualities of high yield, thick stems, high sugar and ability to grow in the sugar cane areas of north India. Millets: Hybrid maize, jowar and bajra have been successfully developed in India. Hybrid breeding have led to the development of several high yielding varieties resistant to water stress. Q Source: Page 173: Chapter 9: NCERT XIIth: Biology

33 Correct Answer : D Justification: Statement 1: Madhubani painting/Mithila painting was traditionally created by the women of the Brahman, Dusadh and Kayastha communities in Mithila region in Nepal and India. So, 1 is incorrect. Statement 2: Originally the paintings depicted an assembly of symbolic images of the lotus plant, the bamboo grove, fishes, birds and snakes in union. These images represented fertility and proliferation of life. So, 2 is incorrect too. Statement 3: Madhubani painting received official recognition in 1975, when the President of India gave Padma Shri award to Jagdamba Devi, of Jitwarpur village near Madhubani. In 1981 Sita Devi was awarded by Padma Shri , In 1990, Ganga Devi of Mithila was awarded Padma Shri. Other painters, Mahasundari Devi (2008), Godavari Dutt, Bharti Dayal and Bua Devi were also given National award. So, 3 is incorrect. Q Source:Page 97: Chapter 9: NCERT XIth: Living Craft Traditions of India

34 Correct Answer : A Learning: Single-cell protein (SCP) refers to edible unicellular microorganisms. The biomass or protein extract from pure or mixed cultures of algae, yeasts, fungi or bacteria may be used as an ingredient or a substitute for protein-rich foods, and is suitable for human consumption or as animal feeds. Benefits: Whereas industrial agriculture is marked by a high water footprint, high land use, biodiversity destruction, general environmental degradation and contributes to climate change by emission of a third of all greenhouse gases, production of SCP does not necessarily exhibit any of these serious drawbacks. As of today, SCP is commonly grown on agricultural waste products, and as such inherits the ecological footprint and water footprint of industrial agriculture. However, SCP may also be produced entirely independent of agricultural waste products through autotrophic growth. Q Source:Page 176: Chapter 10: NCERT XIIth: Biology

35 Correct Answer : A Learning:An expert committee under Dhirendra Singh was constituted in May 2015. The Committee was tasked to evolve a policy framework to facilitate 'Make in India' in defence manufacturing and align the policy evolved with DPP-2013; and to suggest the requisite amendments in DPP-2013 to remove the bottlenecks in the procurement process and also simplify/rationalise various aspects of the defence procurement. Major changes are: Government will fund private R&D for building a defence technology base in the country for which various norms have been stipulated. Under it companies are eligible for up to 90 percent funding. Addition of new category to promote domestic manufacturing, including government funding. The funding will be mainly in technology development for Research & Development (R&D) and recognition of the Micro, Small and Medium Enterprises (MSME). New category Indigenously Designed, Developed and Manufactured (IDDM) platforms have been added and it will be the priority route for procurements. Q Source:http://www.thehindu.com/news/national/fillip-to-make-in-india-in-defence-purchases/article8093796.ece

36 Correct Answer : C Justification: Statement 1: The Gonds are among the largest tribes in Central India, numbering about 4 million. Though predominantly centered in Madhya Pradesh, they are present in significant numbers in the states of Madhya Pradesh, , , and . So, 1 is wrong. Statement 2: The Gonds paint their walls with vibrant depictions of local flora, fauna and gods such as Marahi Devi and Phulvari Devi (Goddess Kali). Traditionally made on festive occasions such as Karwa Chauth, Diwali, Ashtami and Nag Panchmi, Gond painting depicts various celebrations, rituals and man's relationship with nature. Statement 3: The Gonds or the Koiture are a heterogeneous group spreading over large areas from the Godavari gorges in the south to the Vindhya Mountains in the north. In Madhya Pradesh, they inhabited the dense forests of the Vindhyas, Satpura and Mandla in the Narmada region. So, 3 is wrong. Learning: Gond art is very important and will be covered in later tests. Q Source: Page 104 (Box): Chapter 9: NCERT XIth: Living Craft Traditions of India

37 Correct Answer : C Learning: IEA is an inter-governmental organization established in 1974 as per framework of the Organisation for Economic Co-operation and Development (OECD). OES is an intergovernmental collaboration between countries which operates under framework established by the IEA in 2001. By becoming a member India will have access to advanced Research & Development (R&D) teams and technologies across the world. India will also become partner in developing test protocols along with other countries which will in turn help in testing Indian prototypes as per international requirements and norms. Earth System Science Organisation - National Institute of Ocean Technology (ESSO-NIOT) under the aegis of Union Ministry of Earth Sciences will be the nodal agency for the membership. It can also help India to leverage benefits from joint cooperative programs with institutes of member countries could be taken up. http://pmindia.gov.in/en/news_updates/cabinet-approves-india-joining-the-international-energy-agency-ocean-energy-systems/

38 Correct Answer : D Justification: Statement 1: The land revenue department was well organized. It was called as puravuvarithinaikkalam. All lands were carefully surveyed and classified for assessment of revenue. Statement 2: The Cholas had an excellent system of administration. The emperor or king was at the top of the administration. They undertook royal tours to increase the efficiency of the administration. There was elaborate administrative machinery comprising various officials called perundanam and sirudanam. Statement 3 and 4: Besides land revenue, there were tolls and customs on goods taken from one place to another, various kinds of professional taxes, dues levied on ceremonial occasions like marriages and judicial fines. Q Source:Page 146: Chapter 13: Class XIth: TamilNadu History Textbook

39 Correct Answer : A Learning: Chhau is a style performed exclusively by men from the triangular area where Bihar, Bengal and Orissa meet. This is the tribal belt of India home to the tribal groups of Bhulya, Santhals, Mundas, Hos and Oraons. In the olden days the powerful Ganga and Gajapati rulers of Orissa extended their territory from the river Ganges in the north to the Godavari in the south with the help of a vast army of valiant Paikas. They were not in the regular payroll of the army, but received huge land grants from the kings and the chieftains. They formed the rank of a peasant-militia. Though agriculture was their main occupation they used to keep themselves prepared by regular practice and training in war techniques. Q Source:Page 114: Chapter 10: NCERT XIth: Living Craft Traditions of India

40 Correct Answer : A Learning:Many industries benefit from nearness to a leader-industry and other industries. These benefits are termed as agglomeration economies. Savings are derived from the linkages which exist between different industries. These factors operate together to determine industrial location. Q Source: Page 47: Chapter 6: NCERT XIIth: Fundamentals of Human Geography

41 Correct Answer : A Learning:The mashak is a type of bagpipe found in Northern India and parts of Pakistan and Afghanistan. It is usually played by the Dholis as accompaniment to popular folk melodies. The pipe was associated with weddings and festive occasions. In India it is historically found in Rajasthan and Uttar Pradesh. This bagpipe uses single reeds, and can be played either as a drone or as a melody instrument. Q Source:Page 120: Chapter 10: NCERT XIth: Living Craft Traditions of India

42 Correct Answer : D Learning:In animals, the organism, if unable to migrate, might avoid the stress by escaping in time. The familiar case of bears going into hibernation during winter is an example of escape in time. Some snails and fish go into aestivation to avoid summer-related problems-heat and dessication. Under unfavourable conditions many zooplankton species in lakes and ponds are known to enter diapause, a stage of suspended development. Q Source:Improvisation: Page 225: Chapter 13: NCERT XIIth: Biology

43 Correct Answer : A Learning:The Hindu princes of north India formed a confederacy under the command of Prithiviraj Chauhan. Prithviraj defeated Ghori in the battle of Tarain near Delhi in 1191 A.D. In the ensuing Second Battle of Tarain in 1192, Muhammad Ghori thoroughly routed the army of Prithiviraj, who was captured and killed. The second battle of Tarain was a decisive battle. It was a major disaster for the Rajputs. Their political prestige suffered a serious setback. The whole Chauhan kingdom now lay at the feet of the invader. The first Muslim kingdom was thus firmly established in India at Ajmer and a new era in the began. Q Source:Page 168: Chapter 15: Class XIth: TamilNadu History Textbook

44 Correct Answer : A Learning:The Stand-up India is component of Start-up India, Stand up India slogan anchored by Department of Financial Services (DFS) to encourage greenfield enterprises by Women and SC/ST entrepreneurs. It will refinance window through Small Industries Development Bank of India (SIDBI) with an initial amount 10,000 crore rupees. Under it, a credit guarantee mechanism would be created through the National Credit Guarantee Trustee Company (NCGTC). It will be the operating agency for the loan. It will provide handholding support to the borrowers at both pre loan stage and during operations stage. The overall intent is to leverage the institutional credit structure to reach out to this under-served population. For this margin money of the composite loan will be up to 25%. Q Source:http://pib.nic.in/newsite/PrintRelease.aspx?relid=134220 45 Correct Answer : D Learning:The reign of Firoz Tughlaq was more notable for his administration. He strictly followed the advice of the ulemas in running the administration. He pleased the nobles and assured hereditary succession to their properties. Thus the iqta system was not only revived but also it was made hereditary. As per the Islamic law he levied the taxes. Jiziya was strictly imposed on non- Muslims. He was the first Sultan to impose irrigation tax. But at the same time he dug irrigation canals and wells. Q Source:Page 185: Chapter 16: Class XIth: TamilNadu History Textbook

46 Correct Answer : A Concept:Antibiotic resistance occurs when an antibiotic has lost its ability to effectively control or kill bacterial growth; in other words, the bacteria are "resistant" and continue to multiply in the presence of therapeutic levels of an antibiotic. Justification:Statement 1: The evolution of resistant strains is a natural phenomenon that occurs when microorganisms replicate themselves erroneously or when resistant traits are exchanged between them. The use and misuse of antimicrobial drugs accelerates the emergence of drug-resistant strains. So, 1 is correct. Statement 3 and 4: Some bacteria are naturally resistant to certain types of antibiotics. However, bacteria may also become resistant in two ways: 1) by a genetic mutation or 2) by acquiring resistance from another bacterium. Mutations, rare spontaneous changes of the bacteria's genetic material, are thought to occur in about one in one million to one in ten million cells. Some mutations enable the bacteria to produce potent chemicals (enzymes) that inactivate antibiotics, while other mutations eliminate the cell target that the antibiotic attacks. Statement 2: Colonization resistance is the mechanism whereby the intestinal microflora protects itself against incursion by new and often harmful microorganisms. So, 2 is incorrect. Q Source:Improvisation: Page 446: Unit 16: NCERT XIIth: Chemistry

47 Correct Answer : A Justification & Learning:Disinfectants are applied to inanimate objects such as floors, drainage system, instruments, etc. Same substances can act as an antiseptic as well as disinfectant by varying the concentration. For example, 0.2 per cent solution of phenol is an antiseptic while its one percent solution is disinfectant. Chlorine in the concentration of 0.2 to 0.4 ppm in aqueous solution and sulphur dioxide in very low concentrations, are disinfectants. So, 1 is correct. The surface, animate or inanimate, does not cause this difference. So, 2 is incorrect. Q Source:Improvisation: Page 448: Unit 16: NCERT XIIth: Chemistry

48 Correct Answer : D Justification & Learning:Main categories of food additives are as follows: Food colours Flavours and sweeteners Fat emulsifiers and stabilising agents Flour improvers - antistaling agents and bleaches Antioxidants Preservatives Nutritional supplements such as minerals, vitamins and amino acids. Except for Nutritional supplements, none of the above additives have nutritive value. These are added either to increase the shelf life of stored food or for cosmetic purposes. Q Source: Page 449: Unit 16: NCERT XIIth: Chemistry

49 Correct Answer : A Learning:The provinces under the Delhi Sultanate were called iqtas. They were initially under the control of the nobles. But the governors of the provinces were called the muqtis or walis. They were to maintain law and order and collect the land revenue. The provinces were divided into shiqs and the next division was pargana. The shiq was under the control of shiqdar. The pargana comprising a number of villages was headed by amil. The village remained the basic unit of the administration. The village headman was known as muqaddam or chaudhri. The village accountant was called patwari. Q Source: Page 192: Chapter 17: Class XIth: TamilNadu History Textbook

50 Correct Answer : D Justification: Since it allows specialization of production, (a) is correct. Option (b) is a natural consequence of more producers of the same product or service. Since exporters can tap domestic as well as foreign markets, they can produce more, reduce costs and thus reap economies of scale. So, (c) is also correct. Option (d) is not a valid argument out of the all four. Globalisation along with free trade can adversely affect the economies of developing countries by not giving equal playing field by imposing conditions which are unfavourable. With the development of transport and communication systems goods and services can travel faster and farther than ever before. But free trade should not only let rich countries enter the markets, but allow the developed countries to keep their own markets protected from foreign products. Q Source: Page 85: Chapter 9: NCERT XIIth: Fundamentals of Human Geography

51 Correct Answer : A Justification:Many desert plants have a thick cuticle on their leaf surfaces and have their stomata arranged in deep pits to minimise water loss through transpiration. So, 1 is correct. They also have a special photosynthetic pathway (CAM) that enables their stomata to remain closed during day time. So, 1 is correct. Some desert plants like Opuntia, have no leaves-they are reduced to spinesÐand the photosynthetic function is taken over by the flattened stems. So, 3 is correct. Q Source: Improvisation: Page 225: Chapter 13: NCERT XIIth: Biology

52 Correct Answer : D Learning:Exceptions to MFN principle: For example, countries can set up a free trade agreement that applies only to goods traded within the group- discriminating against goods from outside, for e.g. SAFTA or TPP. Or they can give developing countries special access to their markets, for e.g. agricultural markets. Or a country can raise barriers against products that are considered to be traded unfairly from specific countries, for e.g. guns and ammunition. And in services, countries are allowed, in limited circumstances, to discriminate, as under TRIMS. But the agreements only permit these exceptions under strict conditions. In general, MFN means that every time a country lowers a trade barrier or opens up a market, it has to do so for the same goods or services from all its trading partners - whether rich or poor, weak or strong. Q Source:Page 85: Chapter 9: NCERT XIIth: Fundamentals of Human Geography

53 Correct Answer : C Justification:A kolkhoz was a form of collective farm in the Soviet Union. Kolkhozes existed along with state farms. So, 1 is incorrect. Mixed farming was always practiced in India even before colonialism. So, 2 is also incorrect. British introduced plantation agriculture in tea, coffee, tobacco, indigo etc and commercialized Indian agriculture. So, 3 is correct. Q Source:Page 41: Chapter 5: NCERT XIIth: Fundamentals of Human Geography

54 Correct Answer : A Justification:In our body, cell growth and differentiation is highly controlled and regulated. In cancer cells, there is breakdown of these regulatory mechanisms. Normal cells show a property called contact inhibition by virtue of which contact with other cells inhibits their uncontrolled growth. Cancer cells appear to have lost this property. As a result of this, cancerous cells just continue to divide giving rise to masses of cells called tumors. Q Source:Page 157: Chapter 8: NCERT XIIth: Biology

55 Correct Answer : B Justification & Learning:The census of India, 2011 defines urban settlements as "All places which have municipality, corporation, cantonment board or notified town area committee and have a minimum population of 5000 persons, at least 75 per cent of male workers are engaged in non-agricultural pursuits and a density of population of at least 400 persons per square kilometers are urban. Therefore, (a), (c) and (d) can be discarded. (b) will be the right answer. Q Source:Page 92: Chapter 10: NCERT XIIth: Fundamentals of Human Geography

56 Correct Answer : A Learning:Biofortification - breeding crops with higher levels of vitamins and minerals, or higher protein and healthier fats - is the most practical means to improve public health. Breeding for improved nutritional quality is undertaken with the objectives of improving: (i) Protein content and quality; (ii) Oil content and quality; (iii) Vitamin content; and (iv) Micronutrient and mineral content. The Indian Agricultural Research Institute, New Delhi has also released several vegetable crops that are rich in vitamins and minerals, e.g., vitamin A enriched carrots, spinach, pumpkin; vitamin C enriched bitter gourd, bathua, mustard, tomato; iron and calcium enriched spinach and bathua; and protein enriched beans - broad, lablab, French and garden peas. Q Source:Page 176: Chapter 9: NCERT XIIth: Biology

57 Correct Answer : A Learning:In ancient traditions as well as modern times, snakes are symbols of healing powers and opportunities. The snake animal meaning is powerfully connected to life force and primal energy. In many cultures, it is revered as a powerful totem representing the source of life. In the Hindu regions of Asia the serpent or naga is considered a nature spirit. It is the protector of springs, wells and rivers. Serpents bring rain and thus fertility. But, they can also bring floods and drought. The believe that how the naga treats humanity is representative how we treat the snake and its environment. Serpents also carry the elixir of life and immortality. Q Source: Page 97: Chapter 9: NCERT XIth: Living Craft Traditions of India

58 Correct Answer : A Learning:Movement in search of pastures is undertaken either over vast horizontal distances or vertically from one elevation to another in the mountainous regions. The process of migration from plain areas to pastures on mountains during summers and again from mountain pastures to plain areas during winters is known as transhumance. In mountain regions, such as Himalayas, Gujjars, Bakarwals, Gaddis and Bhotiyas migrate from plains to the mountains in summers and to the plains from the high altitude pastures in winters. Similarly, in the tundra regions, the nomadic herders move from south to north in summers and from north to south in winters. Q Source:Page 33: Chapter 5: NCERT XIIth: Fundamentals of Human Geography 59 Correct Answer : D Justification:Literacy is an indicator of a country's socio-economic development as it reveals the standard of living, social status of females, availability of educational facilities and policies of government. Level of economic development (and not just economic growth) is both a cause and consequence of literacy. So, option (c) is not satisfactory. Total years of schooling may increase for the already literate population. So, (a) is not necessarily correct. In India - literacy rate denotes the percentage of population above 7 years of age, who is able to read, write and have the ability to do arithmetic calculations with understanding. Q Source: Page 19: Chapter 3: NCERT XIIth: Fundamentals of Human Geography

60 Correct Answer : D Concept and Justification:BOD refers to the amount of the oxygen that would be consumed if all the organic matter in one litre of water were oxidised by bacteria. The BOD test measures the rate of uptake of oxygen by micro-organisms in a sample of water and thus, indirectly, BOD is a measure of the organic matter present in the water. The greater the BOD of waste water more is its polluting potential. So, (d) is correct. Q Source:Page 183: Chapter 10: NCERT XIIth: Biology

61 Correct Answer : D LearningBhutan is the only country in the world to officially proclaim the Gross National Happiness (GNH) as the measure of the country's progress. Material progress and technological developments are approached more cautiously taking into consideration the possible harm they might bring to the environment or the other aspects of cultural and spiritual life of the Bhutanese. This simply means material progress cannot come at the cost of happiness. GNH encourages us to think of the spiritual, non-material and qualitative aspects of development. HDI merely consists of education index, longevity index and income index. Q Source:Page 27: Chapter 4: NCERT XIIth: Fundamentals of Human Geography

62 Correct Answer : A Learning:The Ganga Action Plan or GAP was a program launched in 1985 to reduce the pollution load on the river. But the efforts to decrease the pollution level in the river were unsuccessful even after spending nearly 1 billion rupees. In 2014-15 annual budget, the FM announced an integrated Ganga development project titled 'Namami Ganga'. As a part of the program, government of India ordered the shut down of 48 industrial units around Ganga. The program has a budget outlay of Rs. 20,000 crore for the next 5 years. 'Namami Gange' will focus on pollution abatement interventions namely Interception, diversion and treatment of wastewater flowing through the open drains through bio-remediation / appropriate in-situ treatment / use of innovative technologies / sewage treatment plants (STPs) / effluent treatment plant (ETPs); rehabilitation and augmentation of existing STPs and immediate short term measures for arresting pollution at exit points on river front to prevent inflow of sewage etc. Significantly the approach is underpinned by socio-economic benefits that the program is expected to deliver in terms of job creation, improved livelihoods and health benefits to the vast population that is dependent on the river. Q Source:Page 185: Chapter 10: NCERT XIIth: Biology

63 Correct Answer : D Justification & Learning:Population ageing is the process by which the share of the older population becomes proportionally larger. Sub-replacement fertility is a total fertility rate (TFR) that (if sustained) leads to each new generation being less populous than the previous one in a given area. So, (a) is wrong. Demographic trap is combination of high fertility (birth rates) and declining mortality (death rates) in developing countries, resulting in a period of high population growth rate (PGR). But, not majority of people may be in the working age group. So, (c) is not appropriate. Demographic dividend, as defined by the United Nations Population Fund (UNFPA) means, "the economic growth potential that can result from shifts in a population's age structure, mainly when the share of the working-age population (15 to 64) is larger than the non-working-age share of the population (14 and younger, and 65 and older). Q Source:Page 19: Chapter 3: NCERT XIIth: Fundamentals of Human Geography

64 Correct Answer : D Justification:An example of microbial biocontrol agents that can be introduced in order to control butterfly caterpillars is the bacteria Bacillus thuringiensis. So, 1 is correct. Microbial pesticides also include viruses for e.g. entomopathogenic fungi or viruses. So, 2 is correct. A biological control being developed for use in the treatment of plant disease is the fungus Trichoderma. Trichoderma pecies are free-living fungi that are very common in the root ecosystems. They are effective biocontrol agents of several plant pathogens. So, 3 is correct. The very familiar beetle with red and black markings – the Ladybird, and Dragonflies are useful to get rid of aphids and mosquitoes, respectively. So, 4 is also correct. Q Source:Page 187: Chapter 10: NCERT XIIth: Biology

65 Correct Answer : C Learning: reached China in the beginning of the first century A.D. A number of Chinese pilgrims like Fahien and Hiuen Tsang visited India. On the other side, hundreds of Buddhist monks like Gunabhadra, Vajrabothi, Dharmadeva and Dharmagupta visited China. Indian scholars translated many Sanskrit works at the request of Chinese emperors. This contact with China continued even in the thirteenth century when the Mongols established their empire in China. Chinese art had also been influenced by Indian art. Q Source:Page 155: Chapter 14: Class XIth: TamilNadu History Textbook

66 Correct Answer : C Justification: Asexual reproduction preserves the genetic information, while sexual reproduction permits variation. Traditional hybridisation procedures used in plant and animal breeding, very often lead to inclusion and multiplication of undesirable genes along with the desired genes. The techniques of genetic engineering which include creation of recombinant DNA, use of gene cloning and gene transfer, overcome this limitation and allows us to isolate and introduce only one or a set of desirable genes without introducing undesirable genes into the target organism. So, both 1 and 2 are correct. Q Source:Page 194: Chapter 11: NCERT XIIth: Biology

67 Correct Answer : A Learning:The artists compose songs that they sing while they slowly unroll each scene of the painting. Old fabric is pasted on the back of the scroll to make it stronger. These village storytellers travelled from village to village listening to news and passing on information much like television today. The Gujarat earthquake of 2001 and the tsumani of 2004 inspired such singer-artists to present ballads of these natural disasters. Q Source:Page 109: Chapter 9: NCERT XIth: Living Craft Traditions of India

68 Correct Answer : C Learning:The diagram below shows the biome distribution with respect to temperature and precipitation. If you can't see the image, see the Q source. Q Source:Page 220: Chapter 13: NCERT XIIth: Biology

69 Correct Answer : C Learning:Three brain chemicals -- noradrenaline (norepinephrine), serotonin, and dopamine -- are involved in both brain and bodily functions. Noradrenaline and serotonin have been consistently linked to psychiatric mood disorders such as depression and bipolar disorder. Dopamine is commonly linked with the pleasure system of the brain. Disruption to the dopamine system is connected to psychosis and schizophrenia, a severe mental disorder characterized by distortions in reality and illogical thought patterns and behaviors. The brain chemical serotonin is connected to many body functions such as sleep, wakefulness, eating, sexual activity, impulsivity, learning, and memory. Researchers believe that abnormal functioning of brain circuits that involve serotonin as a chemical messenger contribute to mood disorders Q Source: Page 444: Unit 16: NCERT XIIth: Chemistry

70 Correct Answer : A Justification & Learning:If a freshwater fish is put in salt water, the salty sea water would absorb the water from the fish via its gills due to osmosis until it dies. The opposite happens to a saltwater fish placed in fresh water. It instead absorbs too much freshwater through its gills due to reverse osmosis and dies as a result. Some fish can live in both fresh and salt water (salmon, eels, bull sharks for instance) but they require time to adjust between the two. Q Source:Page 222: Chapter 13: NCERT XIIth: Biology

71 Correct Answer : B Learning:It shortens the distance between New York and San Francisco by 13,000 km by sea. Likewise the distance between Western Europe and the West-coast of U.S.A.; and North-eastern and Central U.S.A. and East and South-east Asia is shortened. The economic significance of this Canal is relatively less than that of the Suez. However, it is vital to the economies of Latin America. Q Source:Page 74: Chapter 8: NCERT XIIth: Fundamentals of Human Geography

72 Correct Answer : B Learning:Statement 1: The most well-known northern Indian double skin drum that is used especially for accompanying the old northern Indian Dhrupad style, is the Pakhawaj (simlar to the Mridangam in southern India). Statement 2: It is a drum made out of the wood of the jackfruit tree. It has two sides for playing, made out of leather, and has different kind of sounds on each side. The maddalam is a heavy instrument which is hung around the waist of the person playing, and the player stands all the while to perform. The maddalam is a vital instrument in traditional Kerala percussion ensembles like Panchavadyam, Keli and Kathakali orchestra. Statement 3: Keekli is a dance in the rajouri district of J&K. The main instruments played with these folk songs and folk dances are algoza, Dokra, Sarnai, Glass, Thumbak Nari, Sargi, flute, Dholak and Dhol etc. Q Source:Page 122: Chapter 10: NCERT XIth: Living Craft Traditions of India

73 Correct Answer : C Justification:Statement 1: Main problem that appears in the use of detergents is that if their hydrocarbon chain is highly branched, then bacteria cannot degrade this easily. Slow degradation of detergents leads to their accumulation. So, 1 is correct. Therefore detergents with straight chain of hydrocarbons are preferred over branched chain as the latter are non-biodegradable and consequently cause environmental pollution. Statement 2: Effluents containing such detergents reach the rivers, ponds, etc. These persist in water even after sewage treatment and cause foaming in rivers, ponds and streams and their water gets polluted. These days the branching of the hydrocarbon chain is controlled and kept to the minimum. Unbranched chains can be biodegraded more easily and hence pollution is prevented. Q Source:Page 452: Unit 16: NCERT XIIth: Chemistry

74 Correct Answer : A Concept:Both the species benefit in mutualism and both lose in competition in their interactions with each other. In both parasitism and predation, only one species benefits (parasite and predator, respectively) and the interaction is detrimental to the other species (host and prey, respectively). The interaction where one species is benefitted and the other is neither benefitted nor harmed is called commensalism. In amensalism on the other hand one species is harmed whereas the other is unaffected. Predation, parasitism and commensalism share a common characteristic– the interacting species live closely together. Justification:Option (b) is commenalism where one species derives benefit from the other, while the other does not. Mango tree supports orchid, but orchid does not given any visible benefit to the mango tree. Option (c) is similar to parasitism. Option (d) is the 'Competition' nature of interaction. Q Source:Improvisation: Page 232: Chapter 13: NCERT XIIth: Biology

75 Correct Answer : A Learning:Aihole was the first capital of Chalukyas where they built numerous temples dating back to the 6th century CE. Many inscriptions found at Aihole, but the inscription which found at Meguti Temple popularly known as Aihole inscription, which has the significance in the history of India, witnessed for the many historical events of Chalukyas. The inscription written in Sanskrit and it is in script. There is a mention about the defeat of Harshavardhana by Pulikeshi II. And there is a mention about the victory of Chalukyas on Pallavas, also mentioned about the shifting of the capital from Aihole to by Pulikeshi. There is mention about the poet . The Aihole inscription issued by Pulakeshin II gives the details of his reign. Q Source:Page 132: Chapter 12: Class XIth: TamilNadu History Textbook

76 Correct Answer : B Justification:Lower population growth may not necessarily show higher levels of economic development. It can be due to high death rate too. So, (a) is incorrect. Option (c) is an incorrect statement. Natural capital exists in the nature, e.g. forests, lakes, land etc. Option (d) is not appropriate, as higher increase in population may reduce per capita human capital, and not total human capital. Q Source:Page 14: Chapter 2: NCERT XIIth: Fundamentals of Human Geography

77 Correct Answer : C Justification & Learning: The Rashtrakuta Empire was divided into several provinces called rashtras under the control of rashtrapatis. They were further divided into vishayas or districts governed by vishayapatis. The next subdivision was bhukti consisting of 50 to 70 villages under the control of bhogapatis. These officers were directly appointed by the central government. So, 1 is correct. The village administration was carried on by the village headmen. However, the village assemblies played a significant role in the village administration. So, 2 is correct. Q Source:Page 136: Chapter 13: Class XIth: TamilNadu History Textbook

78 Correct Answer : C Justification:Many other routes have natural harbours, for e.g Pacific route. So, R is incorrect. The North Atlantic sea route links North-eastern U.S.A. and Northwestern Europe, the two industrially developed regions of the world. The foreign trade over this route is greater than that of the rest of the world combined. One fourth of the world's foreign trade moves on this route. It is, therefore, the busiest in the world and otherwise, called the Big Trunk Route. Both the coasts have highly advanced ports and harbour facilities. This sea route passes through the heart of the Old World and serves more countries and people than any other route. Port Said, Aden, Mumbai, Colombo and Singapore are some of the important ports on this route. Q Source:Page 73: Chapter 8: NCERT XIIth: Fundamentals of Human Geography

79 Correct Answer : A Justification & Learning:A bioreactor may refer to any manufactured or engineered device or system that supports a biologically active environment. This process can either be aerobic or anaerobic. So, 1 is correct. A bioreactor may also refer to a device or system meant to grow cells or tissues in the context of cell culture. These devices are being developed for use in tissue engineering or biochemical engineering. So, 2 is correct. A photobioreactor (PBR) is a bioreactor which incorporates some type of light source. Virtually any translucent container could be called a PBR, however the term is more commonly used to define a closed system, as opposed to an open tank or pond. Photobioreactors are used to grow small phototrophic organisms such as cyanobacteria, algae, or moss plants. So, 3 is correct. Q Source: Page 204: Chapter 11: NCERT XIIth: Biology

80 Correct Answer : B Justification:Cyanobacteria are autotrophic microbes widely distributed in aquatic and terrestrial environments many of which can fix atmospheric nitrogen. So, 1 is incorrect. Anabaena, Nostoc, Oscillatoria, etc are important cynaobacteria. In paddy fields, cyanobacteria serve as an important biofertiliser. Blue green algae also add organic matter to the soil and increase its fertility. So, 2 and 3 are correct. Currently, in our country, a number of biofertilisers are available commercially in the market and farmers use these regularly in their fields to replenish soil nutrients and to reduce dependence on chemical fertilisers. Q Source:Page 188: Chapter 10: NCERT XIIth: Biology

81 Correct Answer : D Justification: Antacids only treat the symptoms of the diseases like Ulcer, not their cause. So, (a) is wrong. Bile is a digestive juice that is secreted by the liver (not stomach) and stored in the gallbladder. So, (b) would be wrong. Lower pH means acidity of the stomach is increased, so (c) can't be the answer. Learning: An antacid is a substance which neutralizes stomach acidity, which in turn relieves heartburn. Antacids are available over the counter and are taken by mouth to quickly relieve occasional heartburn, the major symptom of gastroesophageal reflux disease. Treatment with antacids alone is symptomatic and only justified for minor symptoms. Q Source: Page 433: Unit 16: NCERT XIIth: Chemistry 82 Correct Answer : A Learning:Market reforms: Alauddin Khalji established four separate markets in Delhi, one for grain; another for cloth, sugar, dried fruits, butter and oil; a third for horses, slaves and cattle; and a fourth for miscellaneous commodities. Each market was under the control of a high officer called Shahna-i-Mandi. The supply of grain was ensured by holding stocks in government storehouses. Regulations were issued to fix the price of all commodities. Land reforms: Apart from market reforms, Alauddin Khalji took important steps in the land revenue administration. He was the first Sultan of Delhi who ordered for the measurement of land. Even the big landlords could not escape from paying land tax. Land revenue was collected in cash in order to enable the Sultan to pay the soldiers in cash. His land revenue reforms provided a basis for the future reforms of Sher Shah and Akbar. Q Source: Page 180: Chapter 16: Class XIth: TamilNadu History Textbook

83 Correct Answer : D Justification: Temperature is one of the most important ecologically relevant factors. For e.g. mango trees do not and cannot grow in temperate countries like Canada and Germany where temperatures are low. So, 1 is incorrect. Tuna fish are rarely caught beyond tropical latitudes in the ocean. So, 2 would be incorrect. Snow leopards are found in very cold climates, for e.g. in northwestern Himalayas. So, 3 would be incorrect. Q Source: Page 220: Chapter 13: NCERT XIIth: Biology

84 Correct Answer : B Learning: Diwan-i-Khairat was created to take care of orphans and widows. Free hospitals and marriage bureaus for poor Muslims were also established by Firoz. However he increased the number of slaves by capturing the defeated soldiers and young persons. In his regime the number of slaves had increased to one lakh eighty thousand. When Firoz died in 1388 the struggle for power between the Sultan and the nobles started once again. His successors had to face the rebellion of the slaves created by Firoz. Q Source: Page 185: Chapter 16: Class XIth: TamilNadu History Textbook

85 Correct Answer : B Learning: Ortho-sulphobenzimide, also called saccharin, is the first popular artificial sweetening agent. It has been used as a sweetening agent ever since it was discovered in 1879. It is about 550 times as sweet as cane sugar. It is excreted from the body in urine unchanged. Alitame is high potency sweetener, although it is more stable than aspartame, the control of sweetness of food is difficult while using it. Sucrolose is trichloro derivative of sucrose. Its appearance and taste are like sugar. It is stable at cooking temperature. It does not provide calories. Q Source: Page 450: Unit 16: NCERT XIIth: Chemistry

86 Correct Answer : C Justification: Statement 1: Sufism stressed the elements of love and devotion as effective means of the realisation of God. Love of God meant love of humanity and so the Sufis believed service to humanity was tantamount to service to God. While orthodox Muslims emphasise external conduct, the Sufis lay stress on inner purity. Statement 2: According to them one must have the guidance of a pir or guru, without which spiritual development is impossible. This is because the path of devotion can be very tough for a novice follower. Sufism also inculcated a spirit of tolerance among its followers. Other ideas emphasised by Sufism are meditation, good actions, repentance for sins, performance of prayers and pilgrimages, fasting, charity and suppression of passions by ascetic practices. Q Source:Page 200: Chapter 18: Class XIth: TamilNadu History Textbook

87 Correct Answer : C Learning:PMFBY will replace the existing two schemes National Agricultural Insurance Scheme as well as Modified NAIS which have had some inherent drawbacks. Farmers' premium has been kept at a maximum of 2 per cent for foodgrains and oilseeds and up to 5 per cent for horticulture/cotton crops. According to sources, PMFBY will increase the insurance coverage to 50 per cent of the total crop area of 194.40 million hectare from the existing level of about 25-27 per cent crop area. The expenditure is expected to be around Rs 9,500 crore. In PMFBY, there will not be a cap on the premium and reduction of the sum insured Q Source:http://www.thehindu.com/news/national/govt-approves-new-crop-insurance-plan-for-farmers/article8102137.ece

88 Correct Answer : D Learning:Statement 1: Prehistoric paintings in rock shelters and walls of caves show early life and activities of human society. Statement 2: Buddhist viharas or monasteries and chaitya or prayer halls in Ajanta in Maharashtra, Alchi monastery in Ladakh, and in Bagh in Madhya Pradesh have murals depicting the life of Buddha and other religious stories. Statement 3: Wall paintings can be found in the temples of Kailashnath Temple of in Tamil Nadu. Recently wall paintings were found in the Brihadesvara Temple of Thanjavur, Tamil Nadu. Early examples of Jaina paintings were found in Sittanavasal in Pudukottai District of Tamil Nadu. At Virubhadra Temple in Lepakshi are examples of the Andhra style of mural painting. Q Source:Page 105: Chapter 9: NCERT XIth: Living Craft Traditions of India

89 Correct Answer : C Learning:Kerala state was able to achieve 100 per cent primary education through its flagship primary education equivalency drive Athulyam Scheme. The programme was meant to provide primary school education to people aged between 15 and 50. It was launched in 2013. The state level literacy mission aimed at educating those who had been unable to complete their primary education up to fourth standard. The mission mainly focused on deprived sections viz. the scheduled caste, scheduled tribe, physically, financially weak and mentally disabled among others. The second stage of the scheme was Mission 676 that also aimed at targeting those who had dropped out of the formal education process. Q Source: http://indiatoday.intoday.in/education/story/kerala-primary-education/1/568315.html

90 Correct Answer : D Justification: Demographic transition theory can be used to describe and predict the future population of any area. The theory tells us that population of any region changes from high births and high deaths to low births and low deaths as society progresses from rural agrarian and illiterate to urban industrial and literate society. These changes occur in stages which are collectively known as the demographic cycle. So, both 1 and 2 are incorrect. Q Source: Page 15: Chapter 2: NCERT XIIth: Fundamentals of Human Geography

91 Correct Answer : C Learning:The Employees' Provident Fund Organisation (EPFO) has won the National Award on e-Governance 2015-16 for launching the Universal Account Number (UAN). UAN will help to reduce the workload of the EPFO to a great extent as it receives over 12 lakh claims for transfer of PF account on changing of jobs by its subscribers. It administers a compulsory contributory Provident Fund Scheme (1952), Pension Scheme (1995) and Insurance Scheme (1976). It is one of the largest social security organisations in India in terms volume of financial transactions undertaken and number of covered beneficiaries. Q Source:http://www.thehindu.com/news/national/epfo-bags-national-award-on-egovernance/article8098318.ece

92 Correct Answer : B Learning: The IMD has never used the term "drought" in its forecasts and has maintained that declaring droughts was the prerogative of States. The agency had several definitions of drought: meteorological, hydrological and agricultural, and it was quite possible for a State to have a meteorological drought- 90 per cent shortfall of the average monsoon rainfall Ñ but not suffer an agricultural drought Ñif the shortfall didn't affect more than 20 per cent of the State's area. Refer to the image below (or in the Q source) for the new definitions. Q Source:http://www.thehindu.com/news/national/no-more-droughts-in-india-says-imd/article8093638.ece

93 Correct Answer : A Learning:The masterpieces of Kerala mural art include: the Temple in Ettumanoor, the Ramayana murals of Mattancherry Palace and Vadakkumnatha kshetram. The "Gajendra Moksham" mural painting in the Krishnapuram Palace near Kayamkulam, the Anantha Shayanam mural painting in the Pallikurup Mahavishnu Temple, Mannarkkad Palakkad District and the mural paintings in the sanctom of Padmanabha temple at Thiruvananthapuram are very famous. The murals of Thirunadhikkara Cave Temple (now ceded to Tamil Nadu) and Tiruvanchikulam are considered the oldest relics of Kerala's own style of murals. Q Source:Page 104: Chapter 9: NCERT XIth: Living Craft Traditions of India

94 Correct Answer : A Justification:Statement 1: India has more than 50% of its population below the age of 25 and more than 65% below the age of 35. It is expected that, in 2020, the average age of an Indian will be 29 years, compared to 37 for China. Statement 2: But it recognises many of the tribal groups as Scheduled Castes and Tribes. Statement 3: Its wrong. High density states/UTs are Delhi, Chandigarh, WB, Bihar, Pondicherry etc. https://en.wikipedia.org/wiki/Demographics_of_India#Structure_of_the_population_.5B62.5D Q Source:Page 15: Chapter 2: NCERT XIIth: Fundamentals of Human Geography

95 Correct Answer : A Justification & Learning: In the wake of the two India-Pakistan wars in 1965 and 1971, there was migration of people from India to Pakistan. The properties and companies of such migrated persons (acquired Pakistani nationality) were confiscated by Union Government under the Defence of India Rules framed under Defence of India Act. These enemy properties were vested in the Custodian of Enemy Property for India by the Union Government. In the Tashkent Declaration (signed in 1966), a clause was included to discuss the return of the assets and property taken over by either side in connection with the conflict. However, Pakistan Government had disposed of all such enemy properties in their country in 1971. Enemy Property Act, 1968: provided for the continuous vesting of enemy property in the custodian. The possession of enemy properties spread across many states in the country vest with the Union Government through the Custodian of Enemy Property for India. The recent ordinance bridges the gaps in the earlier laws to ensure that the enemy properties that have been vested in the Custodian remain so and they do not revert back to the enemy subject or enemy firm. Q Source:http://pib.nic.in/newsite/PrintRelease.aspx?relid=134302

96 Correct Answer : C Justification: In demographics, the rate of natural increase (RNI) is the crude birth rate minus the crude death rate of a population. Immigration and emigration are counted when total population growth rate is calculated. For e.g. in metropolitan cities in India out of a total growth of 100, nearly 44 comes from natural increase, and not migration. Q Source:Page 9: Chapter 2: NCERT XIIth: Fundamentals of Human Geography

97 Correct Answer : D Justification:A trading bloc can be formed by simply lowering the barriers for members and keeping it intact for non-members. So, 1 is incorrect. It is an economic union that adopts common economic policies. This issue has been dealt in detail in another question on "Free Trade area and Customs Union" in the test. 2 is wrong. Q Source:Improvisation: Page 86: Chapter 9: NCERT XIIth: Fundamentals of Human Geography

98 Correct Answer : A Justification:Due to global economic slowdown and decrease in demand followed by increase in supply (production) has attributed to decrease in global commodity prices especially minerals and oil. So, 1 is correct. Moreover, steel and its related raw material market is also facing a tough time on account of subdued demand and high production which in turn is adversely impacting the prices. However, China and not USA is the world's largest metal consumer. So, 2 would be wrong. It is the fall in demand in China that has subdued overall demand and prices. http://www.business-standard.com/article/pti-stories/govt-abolishes-exportduty-on-iron-ore-pellets-116010501216_1.html

99 Correct Answer : B Learning:The committee was appointed by the SC in 2014 to make recommendations to the BCCI in order to prevent frauds and conflict of interest in cricket administration. Other major recommendations are: BCCI office-bearer's tenure: It should not exceed more than two consecutive terms, and he/she cannot hold two posts at the same time. Membership: Only to team representing the respective states. Each state should have only one vote. Zones: The relevance of different zones should be for the purpose of tournaments and not for the governance of the BCCI and its committees. State Associations: There should be uniformity of structure in the organisation and functioning of state associations on the lines of BCCI. Management: The BCCI management affairs should be done by professionals led by Chief executive officer (CEO). Players Associations: It should be formed for the international as well as for the first class levels. It should be for both men and women teams. The Electoral Officer and Ombudsman: The electoral officer will oversee the election process, while the ombudsman to resolve grievances. Functioning and Transparency: All details and rues of BCCI must be uploaded on the website on BCCI for transparency functioning purpose. Q Source:http://indianexpress.com/article/sports/cricket/lodha-panel-report-top-10-recommendations/

100 Correct Answer : C Justification:The succession to the throne was hereditary. So, (c) is the answer. However, sometimes usurpation to the throne took place as Saluva Narasimha came to power by ending the Sangama dynasty. Learning:The Empire was divided into different administrative units called Mandalams, Nadus, sthalas and finally into gramas. The governor of Mandalam was called Mandaleswara or Nayak. ijayanagar rulers gave full powers to the local authorities in the administration. In the matter of justice, harsh punishments such as mutilation and throwing to elephants were followed. Besides land revenue, tributes and gifts from vassals and feudal chiefs, customs collected at the ports, taxes on various professions were other sources of income to the government. Q Source:Page 212: Chapter 19: Class XIth: TamilNadu History Textbook